Strengthening by defense
I find the greatest difficulties I have with strengthen questions have to do with not understandi...
AndrewArabie on April 25, 2023
  • December 2013 LSAT
  • SEC3
  • Q4
2
Replies
How is answer A wrong?
I feel we don't need to have overdue in our answer because it's already stated that the books are...
Tyler808 on December 23, 2022
  • December 2013 LSAT
  • SEC3
  • Q22
1
Reply
Reason for eliminating D Please
Hi Regarding answer-choice D, LSATMAX provides two explanations for its elimination. The fi...
Mazen on May 16, 2022
  • December 2013 LSAT
  • SEC3
  • Q5
2
Replies
Help
Need help with this question.
sairaj87 on October 2, 2020
  • December 2013 LSAT
  • SEC3
  • Q5
3
Replies
Why A over E
I understand why A can be interpreted as correct but I felt E to be a stronger answer
rwhittle@colgate.edu on August 10, 2020
  • December 2013 LSAT
  • SEC3
  • Q11
1
Reply
explain
please explain
DanielDePasquale on August 6, 2020
  • December 2013 LSAT
  • SEC3
  • Q14
2
Replies
the word "some"
when the premise says the company subcontracts some of their products, does this imply one produc...
kens on April 11, 2020
  • December 2013 LSAT
  • SEC3
  • Q10
1
Reply
still confused
I am still confused as to how I get to answer A. I can eliminate all the other choices, but have ...
kens on April 11, 2020
  • December 2013 LSAT
  • SEC3
  • Q11
1
Reply
Don't quite understand the condition
I don't know how to reach to this: ~GCSP - > ~YPES - > ~FNSC - > ~SE. How do we go from ~GCSP- > ...
kens on April 11, 2020
  • December 2013 LSAT
  • SEC3
  • Q17
1
Reply
Why is B wrong?
Having trouble seeing it, though I see now why A is correct (irrelevance).
isorom19 on April 7, 2020
  • December 2013 LSAT
  • SEC3
  • Q21
1
Reply
D vs. C
Hi, Please explain why D is better than C. Like the student who asked the question below, I don't...
tomgbean on January 12, 2020
  • December 2013 LSAT
  • SEC3
  • Q19
2
Replies
Why is A better than C?
I thought C was irrelevent to the argument so I chose it. What does previous studies have to do w...
tomgbean on January 10, 2020
  • December 2013 LSAT
  • SEC3
  • Q21
1
Reply
Why is D correct?
Answer D states that "The failure to find all of the matter predicted by Einstein's theory should...
Milica Gligic on January 9, 2020
  • December 2013 LSAT
  • SEC3
  • Q2
1
Reply
Confused
Hello, I chose D and I was wondering why the answer choice was B. Thank You in Advance
nb101 on December 26, 2019
  • December 2013 LSAT
  • SEC3
  • Q22
3
Replies
Answer choice A?
Could you tell me why A is not the correct answer? The passage goes from "family and friends" to ...
Kath on December 26, 2019
  • December 2013 LSAT
  • SEC3
  • Q20
2
Replies
Question
Why would this be C and not B? Thanks!
claire_crites on December 26, 2019
  • December 2013 LSAT
  • SEC3
  • Q17
1
Reply
A vs C
I was stuck between A and C. How can I better eliminate C? it took me too long to go between the ...
lerondagates on December 2, 2019
  • December 2013 LSAT
  • SEC3
  • Q7
2
Replies
Why I chose E.
Is my line of thinking okay here? If not, please let me know a better way to approach this. Thank...
lerondagates on December 2, 2019
  • December 2013 LSAT
  • SEC3
  • Q4
2
Replies
Why not C?
How can we eliminate C? Thanks!
lerondagates on December 2, 2019
  • December 2013 LSAT
  • SEC3
  • Q10
2
Replies
Is this a typo? Has Not Adversely
The passage begins by saying, "New evidence indicates that recent property development bordering ...
DDL on November 29, 2019
  • December 2013 LSAT
  • SEC3
  • Q25
3
Replies